Mike wants to buy a used car for $3200. He has$400 in savings and can save another $250 each month. How many months will it take him to be able to buy this car?

Answers

Answer 1

Answer:

12 months

Step-by-step explanation:

mental math


Related Questions

find the area of abc
6m 20m 60m

Answers

Answer:

The area of ABC is 60m sq.

Step-by-step explanation:

Area of triangle = 1/2×base×height

so, Area=1/2×20×6

Area= 60m sq.

4 of 204 of 20 Questions




Question 4
What is the equation of the line that is parallel to y−5=−13(x+2) and passes through the point (6,−1)?

Answers

Answer:

y= -13x+77

Step-by-step explanation:

Parallel lines do not share any points but have the same slope.

First, put the equation y-5=-13(x+2) into slope-intercept form.

y= -13x-21

Slope= -13x

Y-Intercept= (0,-21)

Now that we know the slope...

y-(-1)=-13(x-6)

becomes

y+1=-13x+78

y-1=-13x+78-1

y=-13x+77

Olivia has 4 2/3 yards of fabric to make scarves. She needs 3/4 yards for one scarf. How many
scarves can she make?​

Answers

Answer:

6 scarves

Step-by-step explanation:

So we know that 3/4 yd. = 1 (scarf)

We have 4 2/3 material to make the scarves

=> convert to an improper fraction 4 2/3 = 14/3

=> Divide material by needed amt.

=> 14/3 / 3/4 = 14/3 x 4/3

=> 14/3 x 4/3 = 56/9

56/9 = 6 2/9

But 6 2/9 is not our answer. Since we need a full amt. of scraves, we round down to our final answer of 6 scarves.

Hope this helps!

For the rational equation,[tex]\frac{x^{2}+5x+6}{x+3}=1[/tex] , what is a valid value of x?

Answers

Answer:

-3 or -1 is a valid value for x

Step-by-step explanation:

We start by cross multiplying;

So the expression becomes;

x^2 + 5x + 6 = 1(x + 3)

x^2 + 5x + 6 = x + 3

x^2 + 5x -x + 6-3 = 0

x^2 + 4x + 3 = 0

x^2 + x + 3x + 3 = 0

x(x + 1) + 3(x + 1) = 0

(x + 3)(x + 1) = 0

x + 3 = 0 or x + 1 = 0

x = -3 or x = -1

The parent council is in charge of making lemonade for field day.They purchased 19 bags of lemon.each bag has 24 lemons.The recipe says that a gallon of lemonade will require 8 lemons.they will be able to pour 12 cups of lemonade from each gallon that they make.How many cups of lemonade will the parent council be able to serve?

Answers

Answer:

684

Step-by-step explanation:

19 x 24 = 456    

456 divided by 8 = 57

57 x 12 = 684

A wooden jewelry box has the shape of a prism with a regular hexagonal base of 85.3 in2. The sides of the hexagonal base are all 5.73 inches. If the height of the box is 18.10 inches, what is the surface area of the wood used to make the jewelry box?

Answers

Answer:

792.9 in²

Step-by-step Explanation:

Given:

Area of the base of the regular hexagonal prism box (B) = 85.3 in²

Each side length of hexagonal base (s) = 5.73 in

Height of prism box (h) = 18.10 in

Required:

Surface area of the wood used in making the hexagonal prism box

SOLUTION:

Surface area for any given regular prism can be calculated using the following formula: (Perimeter of Base × height of prism) + 2(Base Area)

Perimeter of the hexagonal base of the prism box = 6(5.73) (Note: hexagon has 6 sides.)

Perimeter of base = 34.38 in

Height = 18.10 in

Base area is already given as 85.3 in²

Surface area of the hexagonal prism box [tex] = (34.38*18.10) + 2(85.3) [/tex]

[tex] = 622.278 + 170.6 = 792.878 in^2 [/tex]

Surface area of the wood used in making the jewelry box ≈ 792.9 in²

HELP PLEASE!!
Which best describes the relationship between the lines
with equations -3x – 4y = 1 and -6x – 8y = 2?
A. Perpendicular
B. Neither perpendicular or parallel
C. Parallel
D. Same line

Answers

Answer:

same line

Step-by-step explanation:

its the exact same equation, but the second is multiplied by 2

I promise i will mark as brainiest

Answers

Answer:

A

Step-by-step explanation:

[tex] \frac{2003}{n} = z + \frac{23}{n} [/tex]

where z is an integer

[tex]2003 = nz + 23[/tex]

[tex]nz = 1980[/tex]

[tex]nz = {2}^{2} \times {3}^{2} \times 5 \times 11[/tex]

possible values of n

n= {1980, 990, 660, 495, 396, 330, 198, 180, 165, 132, 99, 90, 60, 45, 36, 30, 20, 18, 15, 5, 3}

cardinality of n=21

What are the lower quartile, upper quartile, and median for this box and
whisker plot?

A) LQ = 22 UQ = 10 Median = 18.5

B) LQ = 10 UQ = 22 Median = 18

C) LQ = 10 UQ = 22 Median = 18.5

D) LQ = 10 UQ = 22 Median = 19​

Answers

Answer:

C

Step-by-step explanation:

Answer:

B

Step-by-step explanation:

The lower quartile range is shown by the bottom of the box which is at 10.

The median is shown in the middle line, which is closer to 18 than 18.5.

The upper quartile range in the end of the box, which is at 22!

(You can also look at the picture attached if that helps.)

A rectangle's perimeter is equal to 27 plus its width. The length of the rectangle is four times its width. What is the
width of the rectangle in units?

Answers

Answer:

P = 3

Step-by-step explanation:

L = 4*w

P = 27 + w

P = 2*L + 2*w = 27 + w

2L = 2*(4w) = 8w

Put that into the perimeter formula (the first one.)

P = 8w + 2w = 27 + w

10w = 27 + w                        Subtract w from both sides

9w = 27                                Divide by 9

w = 27 / 9                          

w = 3

You should find the Length just so you can check it.

L = 4w

L = 4*3

L = 12

Check

P = 2L + 2w

P = 2*12 + 2*3 = 30

P = 27 + w

P = 27 + 3

P = 30 and everything checks.

Questions attached below (❁´◡`❁)

Answers

Problem 2

Josh forgot to apply the square root to 16 when he went from [tex](x-3)^2 = 16[/tex] to [tex]x-3 = 16[/tex]

Also, he forgot about the plus/minus.

This is what his steps should look like

[tex]x^2 - 6x - 7 = 0\\\\x^2 - 6x = 7\\\\x^2 - 6x +9= 7+9\\\\(x-3)^2= 16\\\\x-3= \pm\sqrt{16}\\\\x-3= 4 \text{ or } x-3= -4\\\\x= 7 \text{ or } x= -1\\\\[/tex]

There are two solutions and they are x = 7 or x = -1. To check each solution, you plug it back into the original equation

Let's try out x = 7

x^2 - 6x - 7 = 0

7^2 - 6(7) - 7 = 0

49 - 42 - 7 = 0

0 = 0 ... solution x = 7 is confirmed. I'll let you check x = -1

====================================================

Problem 3

We will have

a = 1, b = -4, c = 3

plugged into the quadratic formula below to get...

[tex]x = \frac{-b\pm\sqrt{b^2-4ac}}{2a}\\\\x = \frac{-(-4)\pm\sqrt{(-4)^2-4(1)(3)}}{2(1)}\\\\x = \frac{4\pm\sqrt{4}}{2}\\\\x = \frac{4\pm2}{2}\\\\x = \frac{4+2}{2} \ \text{ or } \ x = \frac{4-2}{2}\\\\x = \frac{6}{2} \ \text{ or } \ x = \frac{2}{2}\\\\x = 3 \ \text{ or } \ x = 1\\\\[/tex]

The two solutions are x = 3 or x = 1. You would check this by plugging x = 3 back into the original expression x^2 - 4x + 3. The result should be zero. The same applies to x = 1 as well.

David is the caption of the school soccer team. Last year, the team won four more games than it lost. If the team won 46 games, how many games did it lose?

Answers

Answer:

C

Step-by-step explanation:

Games lost=Games won - 4

Games lost=46-4=42

Factor 6x2 + 12x - 9 completely.
A
3(2x² + (-3)
+ 4x –
B 6x(x – 7)
3(2x - 3)(x + 1)
D 3(2x + 1) (x – 3)

Answers

Answer:

work is pictured and shown

reduce the following rational expression to the lowest form

[tex]\frac{64x^{5} - 64x}{( 8x^{2} +8) (2x +2) }[/tex]

please answer this. But no spam answers please
hurry

Answers

Answer:

4x(x - 1)

Step-by-step explanation:

Factor the numerator and denominator

64[tex]x^{5}[/tex] - 64x ← factor out 64x from both terms

= 64x([tex]x^{4}[/tex] - 1) ← difference of squares

= 64x(x² - 1)(x² + 1) ← x² - 1 is also a difference of squares

= 64x(x - 1)(x + 1)(x² + 1)

---------------------------------

(8x² + 8)(2x + 2) ← factor out 8 and 2 from each factor

= 8(x² + 1) × 2(x + 1)

= 16(x² + 1)(x + 1)

Then expression can be written as

[tex]\frac{64x(x-1)(x+1)(x^2+1)}{16(x^2+1)(x+1)}[/tex] ← cancel (x² + 1) and (x + 1) on numerator/ denominator

= [tex]\frac{64x(x-1)}{16}[/tex] ← cancel common factor 16 on numerator/ denominator

= 4x(x - 1)

12. Write 0.8 as a fraction,
Pls explain in full detail.

Answers

Answer:

8/10

Step-by-step explanation:

10x10 is 100 8x10 would be 80 so 80/100=8/10

Answer:

8/10 = 4/5

Step-by-step explanation:

0.8 is 8-10th which means 8 divided by 10

reducing 8/10 to its lowest term since 8 and 10 has 2 as common factor, then

(8=2*4)/(10=2*5)

if 2 cancels out, then you are left with 4/5

En una empresa trabajan 60 personas. Usan gafas el 16% de los hombres y el 20% de las mujeres. Si el número total de personas que usan gafas es 11. ¿Cuántos hombres y mujeres hay en la empresa?

Answers

Pregunta completa:

En una empresa trabajan 60 personas. Usan gafas el 16% de los hombres y el 20% de las mujeres. Si el numero total de personas que usan gafas es 11. ¿Cuantos hombres y mujeres hay en la empresa?

Responder:

Hombres = 25

Mujeres = 35

Explicación paso a paso:

Dado lo siguiente:

Número de personas que trabajan en la empresa = 60

Porcentaje de hombres que usan anteojos = 16%

Porcentaje de mujeres que usan anteojos = 20%

Número total de personas que usan anteojos = 11

Suponga, Número de hombres en la empresa = m

Número de mujeres = número total - número de hombres = 60 - m

Por lo tanto,

16% de los hombres = 0,16 m

20% de mujeres = 0,2 (60 - m) = 12 - 0,2 m

Por lo tanto,

0,16 m + 12 - 0,2 m = 11

- 0,04 m = 11 - 12

-0,04 m = - 1

m = 1 / 0.04 = 25

Por tanto, Número de hombres en la empresa = m = 25

Número de mujeres en la empresa = (60 - m) = (60 - 25) = 35 mujeres

Given 4,7,10,13. Calculate the 20th term.​

Answers

Answer:

61

Step-by-step explanation:

to calculate the 20th term you use the formula

Tn=a+(n-1)d where a stands for the first term,n the number of terms and d the common difference.in this case the first term is 4 the common difference is 3 cause they were adding 3 to go to the next term.. therefore the solution will be:

Tn=4+(20-1)3

=4+19×3

=4+57

=61

the 20th term is 61

I hope this helps

the sum of two numbers is 63. one ninth of the first number plus one sixth of the second number is 21. find the numbers

Answers

Answer: x= -189, y =252

Step-by-step explanation:

Let the first number be x and second number be y

so

x + y = 63

x = 63-y

now

1/9 of x + 1/6 of y = 21

x/9 + y/6 = 21

substituting x's value from equation i

(63-y)/9 + y/6 = 21

(378-6y+9y)/54 = 21

378+3y = 1134

3y = 1134-378

so, 3y = 756

so, y = 756/3

so, y = 252

now

x = 63-252

so, x = -189

Find the missing probability. P(A)=720,P(B)=35,P(A∩B)=21100 ,P(A∪B)=?

Answers

P (A U B) = 37/50

7/20+3/5+21/100

The missing probability P(A∪B) will be 37/50.

What is the probability?

Probability refers to a possibility that deals with the occurrence of random events.

The probability of all the events occurring need to be 1.

Given information;

P(A)= 7/20,

P(B)=3/5,

P(A∩B) =21/100 ,

We need to find the missing probability P(A∪B).

We know that

P(A∪B)= P(A) + P(B) + P(A∩B)

P(A∪B) = 7/20 + 3/5 + 21/100

P (A U B) = 37/50

Therefore, the missing probability P(A∪B) will be 37/50.

Learn more about probability here;

https://brainly.com/question/11234923

#SPJ5

Please answer this question now

Answers

Hello!

Answer:

[tex]\huge\boxed{V = 60 m^{3}}[/tex]

Formula for the volume of a triangular pyramid:

V = 1/3(bh)

The base is a triangle, so b = 1/2(b · h)

Solve for the base:

b = 1/2(8 · 5)

b = 1/2(40)

b = 20 m²

Solve for the volume:

V = 1/3(20 · 9)

V = 1/3(180)

V = 60 m³.

Hope this helped you!

Answer:

60 cubic meters

Step-by-step explanation:

I used the formula V = 1/3 AH to figure it out

f(x)=2-3x domain= {-1,0,1,2}

Answers

Answer:

  range = {5, 2, -1, -4}

Step-by-step explanation:

Maybe you want the corresponding range.

  f({-1, 0, 1, 2}) = 2 -3{-1, 0, 1, 2} = 2 +{3, 0, -3, -6} = {5, 2, -1, -4}

what is the lowest common denominator of 4 7 5 2 and 1

Answers

Answer:

140

Step-by-step explanation:

Lowest common denominator 4 , 7 ,5 , 2  and 1

= 4 * 7 * 5

= 140

Answer:

1

Step-by-step explanation:

Since usually common denominator is usually used with fractions (in this case), we can find out it's one because of how no numbers (except for 2 and 4) go into each other without a remainder except for one, so one would be the correct answer

what is the value of 600.79-40.0032+5.01 to the nearest Hundredths​

Answers

Answer:

565.80

Step-by-step explanation:

600.79-40.0032+5.01 =  565.7968

565.7968 to the nearest Hundredths​ =

565.80

Answer:

565.80

Step-by-step explanation:

600.79 - 40.0032 + 5.01

600.79 - 40.0032 = 560.7868

560.7868 + 5.01 = 565.7968

565.7968 to the nearest hundredth = 565.80

How to do this question plz answer me step by step plzz ​

Answers

Answer:

  4 cm

Step-by-step explanation:

At 12 cm depth, the depth is 12/15 = 4/5 of the height of the container.

When the container is tipped on its side, so its height is 5 cm, the depth will still be 4/5 of the height.

  (4/5)(5 cm) = 4 cm

The depth will be 4 cm.

What is 9x - 2 - 13x + 16 simplified ???

Answers

Answer:

Simplifying

9x + -2 + -13x + 16 = 0

Reorder the terms:

-2 + 16 + 9x + -13x = 0

Combine like terms: -2 + 16 = 14

14 + 9x + -13x = 0

Combine like terms: 9x + -13x = -4x

14 + -4x = 0

Solving

14 + -4x = 0

Solving for variable 'x'.

Move all terms containing x to the left, all other terms to the right.

Add '-14' to each side of the equation.

14 + -14 + -4x = 0 + -14

Combine like terms: 14 + -14 = 0

0 + -4x = 0 + -14

-4x = 0 + -14

Combine like terms: 0 + -14 = -14

-4x = -14

Divide each side by '-4'.

x = 3.5

Simplifying

x = 3.5

Step-by-step explanation:

Answer:

-4x+14

Step-by-step explanation:

Collect "Like Terms" (9x and -13) = -4x

-4x - 2 + 16= -4x + 14

Please answer it now in two minutes

Answers

Answer:

x = 84

Step-by-step explanation:

∠ UWV is an angle in a semicircle and is right = 90°

VW = UW  thus Δ UVW is right isosceles, thus

∠ UVW = ∠ WUV = 45° ( sum of angles in triangle ) , thus

x - 39 = 45 ( add 39 to both sides )

x = 84

can someone slove this −8 + x = 3

Answers

Answer:

11

Step-by-step explanation:

Subtract:

-8 + x = 3

+8        +8

--------------

    x = 11

Hope this helped.

Answer:

x = 11

Step-by-step explanation:

−8 + x = 3

Add 8 to both sides

x = 11 ✓

kelly used 2.1 x 10^6 KB of data so far this month. her younger brother joseph used 7 x 10^5 KB of the data so far this month if their share family plan allows them to use 3,000,000 KB of the data each month, how much data usage o they have available for the remainder of this month?

Answers

Answer:

They will 200,000 kb left

Step-by-step explanation:

  2.1 x 10^6=2,100,000     7 x 10^5=700,000

We use the formula V = πr2h to find the volume of a cylinder. Make h the subject of the formula.

Answers

Answer:

V / ( pi r^2)  = h

Step-by-step explanation:

V = pi r^2 h

Divide each side by pi r^2

V / ( pi r^2)  = pi r^2 h / pi r^2

V / ( pi r^2)  = h

Can integers be written as fractions?

Answers

Answer:

Step-by-step explanation:

Yes you can just write them with denominator 1,

So 3 = 3/1 and -6 = -6/1.

Answer:

Yes.

Step-by-step explanation:

ALL real numbers can be written as fractions, and since integers fall under the category of real numbers, it is official that they can be written as fractions.

I am joyous to assist you at any time.

Other Questions
how might q teen's health and social life be affected if the teen tested positive for HIV Describe the assembly process and list five possible materials suitable for use in an assemblage. One of the necessary conditions for a contestable market is that new firms entering the market have a cost advantage over the existing firms.A. TrueB. False Can somebody please give me the definition of tantalizing? I can't figure it out. True or false? If it is false, replace the underlined word with the correct word. (Constant is underlined) The constant term in a polynomial expression is a number that is not multiplied by avariable. Jimmy measured to find the total number of square inches that covered the top of a rectangular table.Which was Jimmy measuring?0A. areaB. circumferenceC. distanceD. perimeterE. volume Halo Kakak Ini Bahasa Inggris Tolong Di Kerjakan Ya Kak Activity 2: Match the words on the left with their definitions on the right. They are underlined.1. chubby a) substances contained in food2. lately b) mature or grow older3. slim c) very beneficial4. avoid d) physical exercise5. work out e) fat6. do wonders f) recently7. age (verb) g) thin8. nutrients h) keep away from Analyzing RhymeRefer to the stanza to answer the questions.Which words rhyme in the first two lines?zed through this rhyme?It's all I have to give,and all anyone needs to live,and to go on living inside,when the world outsideno longer cares if you live or dieremember,I love you.-"I Am Offering This Poem,"Jimmy Santiago Bacaoutside and insidehave and needsgive and live Which woman is LEAST likely to have shed her ethnically proscribed gender roles? A. A woman at a university in Pakistan B. A woman in a city in Saudi Arabia C. A woman in a manufacturing town in China D. A woman in a farming village in Russia E. A woman at a high tech firm in Singapore Which of the following identifies the part of Malthus's theory that was incorrect? A. Food production grew much more quickly than Malthus predicted. B. The population grew much more than Malthus predicted. C. A famine caused food production to fall instead of grow as Malthus predicted. D. A war reduced the population size instead of growing as Malthus predicted. E. The population remained steady and did not grow as Malthus predicted. Which of the following scenarios is supported by the gravity model? A. A Canadian immigrant chooses to move to South Africa over the United States. B. A Canadian immigrant chooses to move to Sudan over the United Kingdom. C. A Chinese immigrant chooses to move to Liechtenstein over Russia. D. A Chinese immigrant chooses to move to Argentina over Spain. E. A Chinese immigrant chooses to move to France over the United States. what does "aww" mean? where does it come from? A 207-kg merry-go-round in the shape of a uniform, solid, horizontal disk of radius 1.50 m is set in motion by wrapping a rope about the rim of the disk and pulling on the rope. What constant force would have to be exerted on the rope to bring the merry-go-round from rest to an angular speed of 0.750 rev/s in 2.00 s What was the significance of Thoreau's comparing the nation's "establishment" with the "households" of its citizens? Can you help me guys convert decimal number into binary numbers (265)10 Polygons EFGH and E'F'G'H' are shown on the following coordinate grid:hyWhat set of transformations is performed on EFGH to form E'F'G'H'? Which is one of the transformations applied to the graph of f(x) = X^2 to change it into the graph of g(x) = -x^2 +16x - 44 Based on its location on the periodic table, which element would be most likely to form a negative ion? A. Bromine (Br) B. Strontium (Sr) C. Potassium (K) D. Magnesium (Mg) 6x = 5y - 2x = 8How many solutions does the system have? fill in the missing. 1:1000=1cm:_?